Тёмный

International Math Olympiad | 2005 Q4 

Michael Penn
Подписаться 300 тыс.
Просмотров 48 тыс.
50% 1

We look at a nice number theory problem from the 2005 International Mathematics Olympiad. Our approach involves Fermat's Little Theorem.
Please Subscribe: ru-vid.com...
Merch: teespring.com/stores/michael-...
Personal Website: www.michael-penn.net
Randolph College Math: www.randolphcollege.edu/mathem...
Randolph College Math and Science on Facebook: / randolph.science
Research Gate profile: www.researchgate.net/profile/...
Google Scholar profile: scholar.google.com/citations?...
If you are going to use an ad-blocker, considering using brave and tipping me BAT!
brave.com/sdp793
Buy textbooks here and help me out: amzn.to/31Bj9ye
Buy an amazon gift card and help me out: amzn.to/2PComAf
Books I like:
Sacred Mathematics: Japanese Temple Geometry: amzn.to/2ZIadH9
Electricity and Magnetism for Mathematicians: amzn.to/2H8ePzL
Abstract Algebra:
Judson(online): abstract.ups.edu/
Judson(print): amzn.to/2Xg92wD
Dummit and Foote: amzn.to/2zYOrok
Gallian: amzn.to/2zg4YEo
Artin: amzn.to/2LQ8l7C
Differential Forms:
Bachman: amzn.to/2z9wljH
Number Theory:
Crisman(online): math.gordon.edu/ntic/
Strayer: amzn.to/3bXwLah
Andrews: amzn.to/2zWlOZ0
Analysis:
Abbot: amzn.to/3cwYtuF
How to think about Analysis: amzn.to/2AIhwVm
Calculus:
OpenStax(online): openstax.org/subjects/math
OpenStax Vol 1: amzn.to/2zlreN8
OpenStax Vol 2: amzn.to/2TtwoxH
OpenStax Vol 3: amzn.to/3bPJ3Bn
My Filming Equipment:
Camera: amzn.to/3kx2JzE
Lense: amzn.to/2PFxPXA
Audio Recorder: amzn.to/2XLzkaZ
Microphones: amzn.to/3fJED0T
Lights: amzn.to/2XHxRT0
White Chalk: amzn.to/3ipu3Oh
Color Chalk: amzn.to/2XL6eIJ

Опубликовано:

 

25 ноя 2020

Поделиться:

Ссылка:

Скачать:

Готовим ссылку...

Добавить в:

Мой плейлист
Посмотреть позже
Комментарии : 144   
@Seamusoboyle
@Seamusoboyle 2 года назад
Just for the record, the problem was phrased differently to this. It asked for all numbers that were relatively prime to all numbers in this sequence. So it was a little harder, as you had to discover that there a member of the sequence that any prime would divide. Source: was there and got 6 out of 7 for this problem.
@gdemrakul2824
@gdemrakul2824 Год назад
that's true, I saw it like that in the book "104 number theory problems"
@Cazolim
@Cazolim 2 года назад
If you're uncomfortable with fields and inverses mod p etc, here's a slight modification that might feel more natural. Call the number 2^n + 3^n + 6^n - 1= a_n. Now, look at 6a_n. Choosing n = p-2 we get 6a_(p-2)= 3*2^(p-1) + 2*3^(p-1)+ 6^(p-1)-6, which we reduce using Fermats little theorem to 3+2+1-6=0. Now we know 6a_(p-2) is divisible by p, but because p is prime and not 2 or 3, this means p divides a_(p-2). This is the same idea, but the solution may be formulated in a more elementary way. Still requires Fermats Little Theorem ofc
@goodplacetostop2973
@goodplacetostop2973 3 года назад
7:40
@LOUAFIMED20
@LOUAFIMED20 3 года назад
Thanks
@mathsamtube2741
@mathsamtube2741 3 года назад
Wow you uploaded three videos in one day. I envy your passion. Hope you get silver button soon.
@shivaudaiyar2556
@shivaudaiyar2556 3 года назад
Thanks so much for such a great content
@xriccardo1831
@xriccardo1831 3 года назад
Yes, thank you professor Penn
@user-ik2kd9mb5t
@user-ik2kd9mb5t 3 года назад
there is general n including for any prime 2 and 3 n = φ(6p) - 1 x = 2^n + 3^n + 6^n - 1 6x = 3×2^φ(6p) + 2×3^φ(6p) + 6^ φ(6p) - 6 = 3 + 2 + 1 - 6 mod 6p 6x is a multiple of 6p so x is a multiple of p upd: it works if p is coprime with 2 and 3
@klausg1843
@klausg1843 2 года назад
What a beautiful solution. Even with a formula for n. Bravo
@udic01
@udic01 3 года назад
I just want to say that I saw your whole number theory lessons, and this is the first one that made me really think and understand about the inverse (-1 "power") numbers.
@abhasjain6381
@abhasjain6381 3 года назад
You don't know how much of a help are you for a student passionate about Maths.
@OuroborosVengeance
@OuroborosVengeance 3 года назад
Thanks Michael Penn. God bless you, this videos are hugely educative
@potspans1470
@potspans1470 3 года назад
There’s a simpler way to prove: 2^(-1) + 3^(-1) + 6^(-1) = 1 ... mod p throughout Without reintroducing p. Using 2x3 = 6 and its multiplicative inverse. You get 2^(-1) + 3^(-1) + 6^(-1) = 3(6^(-1)) + 2(6^(-1)) + 6^(-1) = ( 3+2+1)6^(-1) = 1
@TJStellmach
@TJStellmach 3 года назад
Yes, you can just use the common denominator. It's exactly the same way we know that 1/2+1/3+1/6=1 over the rationals. It only relies on the facts that 6=2*3=2+3+1 and we're operating in a field.
@HienNguyenHMN
@HienNguyenHMN 3 года назад
Your transitions are always top notch.
@beanhwak
@beanhwak 3 года назад
Thank you Mike Penn I could not solve it until I saw you use Fermat Little Theorem.
@andrewmichel2525
@andrewmichel2525 3 года назад
Most fun math channel on youtube. 100k subs soon
@the_dpad
@the_dpad 3 года назад
Huh... if p=3, wouldn't n=2 be a viable solution as well? 2^2 + 3^2 + 6^2 - 1 = 4 + 9 + 36 - 1 = 48
@vinc17fr
@vinc17fr 3 года назад
Since 3 and 6 are divisible by 3, you do not have to compute 3^2 and 6^2. Just ignore them: you get 2^2 − 1 = 3, which is divisible by 3.
@ProjetoEquilatero
@ProjetoEquilatero 3 года назад
Fantástico. Esse cara tem uma visão além do alcance. Parabéns. A aplicação do Pequeno Teorema de Fermat de forma modificada foi incrível. Essa é a verdadeira matemática, o uso de truques que fazem o problema ficar fácil. Uma bela solução de um problema de uma competição tão difícil. Resolveu em pouco tempo. Inacreditável. Parabéns.
@LOUAFIMED20
@LOUAFIMED20 3 года назад
Thanks
@ProjetoEquilatero
@ProjetoEquilatero 3 года назад
Welcome
@calebarulandu3068
@calebarulandu3068 3 года назад
Wow. That's really beautiful. Thanks as always Michael Penn!
@karolchojnacki3924
@karolchojnacki3924 3 года назад
Just found out my algebra professor suggested this problem I can't breathe
@KaraTeUa
@KaraTeUa 3 года назад
Given a right-angled triangle ABC, angle A = 90 degrees. On the BC side, point D is chosen so that the radii of the circles inscribed in the triangular ABD and ACD are the same. It is known that the length of the segment AD is equal to 1 (AD = 1). Find the area of a triangle ABC.
@tomatrix7525
@tomatrix7525 3 года назад
Beautiful. This one was really easy if you knew how to start it.
@AndrejPanjkov
@AndrejPanjkov 3 года назад
Yay, i got this one out. Number theory was never my forte. Essentially same as Michael's proof, but without needing the inverse until the end. The other direction i tried without success was to notice the first three terms are in the expansion of (3^n + 1)(2^n + 1) so the expression can be rewritten as (3^n + 1)(2^n + 1) -2, but then i stalled.
@Tiqerboy
@Tiqerboy 3 года назад
That's exactly what I noticed and tried a few cases for small n, that did yield a number with arbitrary prime factors. But I fail to see how this formula can generate numbers using any value of 'n' where any prime can be a factor. For example for prime 1999, what is the value of n that works?
@RexxSchneider
@RexxSchneider 2 года назад
@@Tiqerboy 1997. The general case showed that 2^(p-2) + 3^(p-2) + 6^(p-2) - 1 ≡ 0 mod p for all p>3. So n =(p-2) is the general solution.
@wasitahmid749
@wasitahmid749 3 года назад
Where is the homework??? Mr GPS, (Good Place to Stop)???
@goodplacetostop2973
@goodplacetostop2973 3 года назад
Last couple of homeworks have been hit-and-miss, probably because the hype is over. So I’m taking a break from them. If they come back, it will be with a new schedule.
@KaraTeUa
@KaraTeUa 3 года назад
Prove that in any right-angled triangle with area 1, the cheviana drawn from the vertex of the right angle and dividing the original triangle into two triangles with equal inscribed circles has length 1.
@j4yb1rd
@j4yb1rd 3 года назад
Doesn't 3 also divide the n=2 case?
@rhythmmandal3377
@rhythmmandal3377 3 года назад
3 divides any n = even number case as 2^2 = 1 mod 3.
@user-ik2kd9mb5t
@user-ik2kd9mb5t 3 года назад
6 doesn't have inverse mod 2 or 3
@beanhwak
@beanhwak 3 года назад
First show 2^-1 = 3^-1 + 6^-1 mod p then 2^(p-2) + 3^(p-2) + 6^(p-2) = 2^-1+3^-1+6^-1=2^-1+2^-2 =2(2^-1) = 1 mod p
@bobzarnke1706
@bobzarnke1706 3 года назад
Since F(p) is a (finite) field, 6^-1 can be factored out of 2^-1 + 3^-1) + 6^-1 directly, giving: 2^(p-2) + 3^(p-2) + 6^(p-2) - 1 = 2^-1 + 3^-1 + 6^-1 - 1 = 6^-1 (3 + 2 + 1) - 1 = 6^-1 * 6 - 1 = 0 (mod p) More generally in any field, a^-1 can be expressed as 1/a and rational expressions can be manipulated as they would be in Q, as long as the denominators aren't 0 (mod p). So, the latter part of the above equation could equally well be expressed as: 2^-1 + 3^-1 + 6^-1 - 1 = 1/2 + 1/3 + 1/6 - 1 = (3 + 2 + 1) / 6 - 1 = 0 (mod p)
@SaveSoilSaveSoil
@SaveSoilSaveSoil 3 года назад
You make it seem so easy.
@beanhwak
@beanhwak 3 года назад
6(2^-1)= (3)(2)(2^-1) = 3 mod p and 6(3^-1)=(2)(3)(3^-1)= 2 mod p, hence by subtraction 6^(2^-1)-6(3^-1) = 3-2 = 1, so 6(2^-1-3^-1)=1 Therefore 2^-1+3^-1 = 6^-1
@nikhileshkrishna889
@nikhileshkrishna889 3 года назад
Extremely good
@copyrightfreemusicforall5563
@copyrightfreemusicforall5563 3 года назад
Thanks sir
@stefanschroder4694
@stefanschroder4694 3 года назад
Muchas gracias ☺😷😷😶
@alexandergertovskiy2126
@alexandergertovskiy2126 Год назад
It's nice. It's very nice!
@RexxSchneider
@RexxSchneider 2 года назад
p=2 works because the expression is always even (even + odd + even - odd) for any value of n. p=3 works with any even n because both 3^n and 6^n ≡ 0 mod 3 and 2^2m ≡ 4^m ≡ 1 mod 3. The simplest case is n=2 when the expression is congruent to 2^2 - 1, which is divisible by 3. For the general case where p >3, we can reduce the amount of working with negative indices by simply considering the expression: (3*2).(2^(p-2) + 3^(p-2) + 6^(p-2) - 1) mod p = (3*2(p-1) + 2*3^(p-1) + 6^(p-1) - 6) mod p ≡ (3*1 + 2*1 + 1 - 6) mod p, using Fermat's Little Theorem ≡ 0 mod p, showing that (2^(p-2) + 3^(p-2) + 6^(p-2) - 1) ≡ 0 mod p, since 6 is not congruent to 0 mod p for p>3. Hence n=(p-2) is a solution to the problem. Although completely equivalent, that just seems a little "cleaner" that the video's explanation.
@ZahlenRMD
@ZahlenRMD 3 года назад
The best Master
@marceliusmartirosianas6104
@marceliusmartirosianas6104 Год назад
2^n+3^n+6^n-1=[[[ 2n+3n+[6n/1=6n]=[ 2n+3n+6n]=[[2+3]=[6+6n]=[ 6+6]=12 tada n=1
@JSSTyger
@JSSTyger 3 года назад
Yes, of course. Why didn't I think of that?
@prime_suntereh2241
@prime_suntereh2241 2 года назад
Sir 1/2, 1/3, 16 are very very special number in this property question was asked in ioqn 2020 of 5 marker
@marcozagaria6696
@marcozagaria6696 3 года назад
Couldn't you more simply use that 1/2 +1/3 + 1/6 -1 is 0 and 0 is a multiple of every number?
@leif1075
@leif1075 3 года назад
How would that evem prove that the reciprocals of those numbers are all.multiples of p, in any case,?
@leif1075
@leif1075 3 года назад
@@angelmendez-rivera351 I know that I meant the sum, I didnt mean individually. I didn't use the word individually. Sorry if I wasn't clear. So my question stands.
@jackw7714
@jackw7714 3 года назад
Whilst it's true that 1/2+1/3+1/6-1 = 0 in Q, that doesn't immediately imply that 2^(-1) + 3^(-1) + 6^(-1) - 1 = 0 mod p, since which elements are in fact the inverse changed. For instance, mod 5, 2^(-1) = 3, 3^(-1) = 2, 6^(-1) = 1^(-1) = 1 [and hence it becomes 3 + 2 + 1 - 1 = 5 = 0, and so it does work, but hopefully this illistrates the issue.]
@vinc17fr
@vinc17fr 3 года назад
@@jackw7714 Wrong, this works in any field where 2 and 3 are invertible (thus 6 is invertible too). Detailed proof: 6·(1/2 + 1/3 + 1/6) = 6·(1/2) + 6·(1/3) + 6·(1/6) = 3·2·(1/2) + 2·3·(1/3) + 1 = 3 + 2 + 1 = 6, then multiply both sides by (1/6), which gives 1/2 + 1/3 + 1/6 = 1.
@DeadlyBlaze
@DeadlyBlaze Год назад
bruh I spent 30 minutes trying to find some kind of relationship with adding modular inverses, and even tried proving that it was possible for at least one to be congruent to -2 (mod p) and I completely forgot that 1/2 + 1/3 + 1/6 = 1.
@lennartbenschop656
@lennartbenschop656 3 года назад
For any even number n: n>0, 2^n+3^n+6^n-1 is a multiple of 3 (and not for any odd n). For any odd number n: 2^n+3^n+6^n-1 is a multiple of 5 (and not for any even n). You can easily demonstrate this by enumerating all cases mod 3 and mod 5 respectively. Hence there will not be an n such that 2^n+3^n+6^n-1 is a multiple of 15. So you can prove that there will not be an n to make 2^n+3^n+6^n-1 a multiple of k for all natural numbers k. So it is true for primes, not for arbitrary natural numbers.
@assroi
@assroi 2 года назад
Amazing proof.
@mcwulf25
@mcwulf25 2 года назад
In fact for p=3 any even value of n works
@lox7182
@lox7182 2 года назад
can someone explain the whole using the ^-1 thing, fractions don't always have the same properties as integers. I feel like I'm missing something here.
@lox7182
@lox7182 2 года назад
oh its the modular inverse thing
@keshavb3128
@keshavb3128 3 года назад
This problem uses a lot of casework.
@prime_suntereh2241
@prime_suntereh2241 2 года назад
Sir pls bring a course for ioqm and inmo pls there need 🙃🙃
@amirb715
@amirb715 3 года назад
very nice :-)
@sonaruo
@sonaruo 3 года назад
Well kick idea that is an EVEN number, and primes except 2 are all odd, so they will be at least 2XpXm where m any natural number
@noeticresearch
@noeticresearch 3 года назад
Maybe I missed it, but you had 4 terms yet when you factored the 6 to the p-2 out of the equation, but later you had only 3 terms. I am trying to understand what happened to the 2 to the p-2 term. I thought that term, once the 6 to the p-2 was factored out, would yield a 1/(3 to the p+2) in the place as one of the four terms. Pardon me if I am wrong. Michael is an excellent mathematican, so I feel completely awkward in pointing out anything that looks curious. :-) Thank you.
@yourlordandsaviouryeesusbe2998
@yourlordandsaviouryeesusbe2998 3 года назад
He had four terms. Notice the -1 outside the parentheses
@user-A168
@user-A168 3 года назад
Good
@adityamohan7366
@adityamohan7366 3 года назад
7:38 so for p primes greater than 3, there is only one solution for n being equal to "p-2" ??
@julianferres
@julianferres 3 года назад
It's clearly not unique, take n=(p-1)*k + (p-2), with k non-negative and it'll will also work
@user-jc2lz6jb2e
@user-jc2lz6jb2e 3 года назад
@@julianferres to add to this, for some primes, the smallest exponent could be smaller than p-2. After all, p-1 in Fermat's little theorem works for ALL numbers coprime to p, but each number could have an exponent that works that is smaller. For example, with p = 11, you can have a = 10 and instead of raising it to 10, you could raise it to 2 and still make it congruent to 1 mod 11. 10^2 = 100 = 99+1 = 1 (mod 11).
@MizardXYT
@MizardXYT 3 года назад
Here are the solutions for p
@computerlover9290
@computerlover9290 2 года назад
What is q in 1:24
@Reliquancy
@Reliquancy 3 года назад
I’m wondering now if for any two different primes p and q there is an n such that p^n -1 is divisible by q
@kobethebeefinmathworld953
@kobethebeefinmathworld953 3 года назад
yes, the Euler's phi function will be the answer
@ireallydontknow3299
@ireallydontknow3299 3 года назад
that follows directly from fermat's little theorem. set n = q - 1. (p doesn't actually need to be prime, btw).
@kobethebeefinmathworld953
@kobethebeefinmathworld953 3 года назад
@@ireallydontknow3299 right, and Euler's phi function is a more general case that even if q is not a prime
@Reliquancy
@Reliquancy 3 года назад
@@ireallydontknow3299 Is there a particular thing you can set n to if neither p and q are necessarily prime? Messing around in maple, it seems like there’s some critical value of n after which any greater n makes p^n-1 a multiple of q for any positive integers p and q. It looks like maybe n=q-1 is an upper bound for the minimum n but idk.
@ireallydontknow3299
@ireallydontknow3299 3 года назад
@@Reliquancy Yes and no. No such thing as "a critical value of n such that _any greater n_ makes p^n - 1 = k * q." What will happen generally (but not always) is that the n will _periodically_ cause p^n - 1 to be divisible by q, which is different than what you proposed there. The only case where it will always be divisible is if p has remainder 1 on division by q. The general form of Fermat's Little Theorem is this: If a and n are coprime integers, then: a^phi(n) == 1 (mod n) (i.e. n divides a^phi(n) - 1) Where phi(n) is the number of integers below n that are coprime with n (no common factors with n).
@webtoon1121
@webtoon1121 3 года назад
Wow Imo in 7 minutes
@jacemandt
@jacemandt 3 года назад
My understanding of multiplicative inverses (mod p) is that, for example, the inverse of 3 (mod 7) is 5, because 3·5 is congruent to 1 (mod 7). But Michael seems to be using a different version of inverses here, where the inverse of 3 is 1/3. My number theory isn't so advanced...can someone help me clear this up?
@jacemandt
@jacemandt 3 года назад
Like...how can anything-like 3^(p-2)-be congruent to 1/3 (mod p) at 5:16?
@thomasstokes9412
@thomasstokes9412 3 года назад
Think of 1/3 for example as being the multiplicative inverse of 3 mod p rather than 1/3 (multiplicative inverse of 3 over the rationals). Michael was working over F_p in this solution and not the rationals.
@ireallydontknow3299
@ireallydontknow3299 3 года назад
The thing is that modular inverses and real reciprocals work pretty much analogously, because they're defined similarly (i.e. 3^-1 (mod p) is the unique residue x such that 3x == 1 (mod p); 1/3 is the unique real number x such that 3x = 1). In a subtle way, 3^-1 (mod p) acts much the same as 1/3. So sometimes people abuse notation a little bit and use 1/3 instead 3^-1. If you see that, it's really just another way of writing 3^-1, but one that acknowledges this "similarity" that it has with 1/3. Don't think of it as actually the real number 1/3. One example of this "similarity" is that, if a, b, c, d, p and q are integers such that: a/b + c/d = p/q then it is also true that ab^-1 + cd^-1 == pq^-1 (mod m) where m is coprime to b, d, q (just so that the inverses actually exist). So you might as well go ahead and write a/b + c/d == p/q (mod m).
@wesleydeng71
@wesleydeng71 3 года назад
It is not necessary to use inverses : 6*(2^(p-2)+3^(p-2)+6^(p-2))(mod p)=3*2^(p-1)+2*3^(p-1)+6^(p-1) (mod p)=3*1+2*1+1=6(mod p) -> 2^(p-2)+3^(p-2)+6^(p-2)=1(mod p)
@thiantromp6607
@thiantromp6607 3 года назад
It proves exactly what we want to prove.
@OlympiadProblemsolving
@OlympiadProblemsolving 5 месяцев назад
but you already have that 2^-1+3^-1+6^-1-1 is congruent to 0 mod p
@mathematicsman7454
@mathematicsman7454 3 года назад
Solve INMO problem
@monikaherath7505
@monikaherath7505 3 года назад
I've never studied university mathematics so I tried to understand this.... After messing around with Fermat's little theorem, I finally understood intuitively why it works when a and p are coprime so I understood that part okay, what completely confounded me was the multiplicative inverse part. I guess I understood what the multiplicative inverse "means" but I have no idea why 1/2 + 1/3 + 1/6 = 1 mod p ; like why would that even work? Why can the multiplicative inverse even be written as fractions and why on earth do they behave as fractions would? I was thinking that maybe it's necessary to have studied some form of number theory at university but that can't be the case because this is an IMO question for high school students. Is all this stuff intuitively obvious to you guys (i.e am I just not smart enough to do this or is this way above my intellectual level) or does this require some form of formal study of number theory to fully grasp? I find it astounding and quite difficult to believe there are people who can just look at this once and understand immediately why it works. I've sat here for around half an hour trying to understand why the multiplicative inverse thing works, why on earth the "inverse" can even be written as a fraction etc. with no success. I tried to look at a few comments and there was talk of fields, but do high school students even know what a field is? I've never even heard of it before. Anyway, I'd really appreciate some advice and resources for people who aren't as smart as IMO people. I really want to understand this (as in really understand deeply and intuitively, not just learn the formula and apply it perfunctorily and mechanically). I'm just in disbelief that there are high school kids my age or younger who can do this, and more importantly, who understand deeply and profoundly how and why all of this works intuitively. It took me over half an hour of deep thinking to even get an intuitive sense of why Fermat's little theorem actually works .... should I just wait til I get to university or is there something I can read to understand this? Or should I just accept this is way beyond my level / IQ. Thanks for your help.
@dansman1729
@dansman1729 3 года назад
My solution wasn't as complicated. You start with 2^n+3^n+6^n-1 congruent to 0 mod p, then factor this to get (2^n+1)(3^n+1) is congruent to 2 mod p. Then the crux move is to set n=p-2, which you can figure out will always work if you play around with small values of p. (2^(p-2)+1)(3^(p-2)+1) congruent to 2 mod p, and we multiply by 36 on both sides. Multiply the 4 in the left factor and the 9 in the right factor to get (2^p+4)(3^p+9) on the LHS, which is congruent to (2+4)(3+9)=72 mod p by Fermat's little theorem, and we're done
@RexxSchneider
@RexxSchneider 2 года назад
@Monika Herath We don't usually write fractions like 1/2 in modular arithmetic because it can sometimes lead us to false conclusions, although these inverses often do behave in a way that is analogous to fractions on the real numbers. Nevertheless we could accept that 1/k mod p means the multiplicative inverse, i.e. the number that when multiplied by k mod p is congruent to 1 mod p. We would more commonly write that inverse as k^(-1). In the case of 1/2 + 1/3 + 1/6 consider the following: Let 2^(-1) + 3^(-1) + 6^(-1) ≡ a mod p. Then we can multiply both sides by 6 = 2*3 ("clearing the denominators"): 2*3*2^(-1) + 2*3*3^(-1) + 2*3*6^(-1) ≡ 2*3a mod p Whatever the value of k^(-1) mod p, it is always true that k*k^(-1) ≡ 1 mod p as that's how we defined it. So we have: 3*1 + 2*1 + 1 ≡ 6a mod p 6 ≡ 6a mod p That shows that a must be ≡ 1 mod p. In other words we have proven that 1/2 + 1/3 + 1/6 ≡ 1 mod p, although using the 2^(-1) etc. notation is preferred. Hope that helps.
@monikaherath7505
@monikaherath7505 2 года назад
@@RexxSchneider Oh my word thankyou so much for explaining it so beautifully and simply. Thanks Rex. Really brilliant explanation, thanks for taking the time to write all of that out.
@tcoren1
@tcoren1 3 года назад
To prove that is equal to 0 couldn't you just do (under mod p) x=(2^-1)+(3^-1)+(6^-1)-1 6x=3+2+1-6=0 x=0 (Since 6≠0 due to the prime being neither 2 nor 3). It really seemed like you were leasing to using this and then you just proved it in a really complex way
@mangai3599
@mangai3599 3 года назад
At 4:25 Yeah..... but at 4:29 What?! I have never done that...!
@LOUAFIMED20
@LOUAFIMED20 3 года назад
Thanks
@bilalabbad7954
@bilalabbad7954 2 года назад
Who have some books in number theory
@mantkcmantk8717
@mantkcmantk8717 3 года назад
This is pretty clumsy... 1/2 + 1/3 + 1/6 = 1 in any field of characteristic not 2 or 3 as it is in the rational numbers, with the same proof.
@foobar5809
@foobar5809 3 года назад
if you write that proof, you will basically expand with 6, which is more or less what he did... so its not that bad
@nzf-kx2qol1g12
@nzf-kx2qol1g12 3 года назад
Wow this video has no dislikes so far.... rn
@wise_math
@wise_math 3 года назад
Dont mention it, now it has dislikes
@atrakchi2
@atrakchi2 3 года назад
👍👍👍👍👍
@vinc17fr
@vinc17fr 3 года назад
First, at 2:15, this is wrong: for p = 2, if n = 0, 2^n and 6^n are not even numbers! Anyway, even n = 0 works. For p = 3, one can do much simpler: assume n ≥ 1, so that 2^n + 3^n + 6^n − 1 ≡ 2^n − 1, then obviously n = 2 works. Concerning p ≥ 5: One has 1/2 + 1/3 + 1/6 = 1 in any field where 2 and 3 are invertible (thus 6 is invertible too, with (1/6) = (1/2)·(1/3)), because the proof (multiply both sides by 6 and simplify the left side) involves only field operations, so that you do not have to consider Q in particular. Hence the result.
@RexxSchneider
@RexxSchneider 2 года назад
I would agree with you, but Michael Penn has adopted the far-from-universally-accepted convention that 0 is not a member of the natural numbers, so he won't consider the n=0 case.
@swapnilgarg6695
@swapnilgarg6695 2 года назад
0 is not a natural number lmao
@vinc17fr
@vinc17fr 2 года назад
@@swapnilgarg6695 0 is a natural number. See the ISO 80000-2 international standard.
@udic01
@udic01 3 года назад
for p=3 => n=2 is much easier...
@GreenMeansGOF
@GreenMeansGOF 3 года назад
n=2 takes care of both cases for p=2 or 3. He should have done that instead.
@udic01
@udic01 3 года назад
@@GreenMeansGOF i am sorry but i disagree. (Although you are right about the value of n=2 right for both cases). I like it the he did.
@leif1075
@leif1075 3 года назад
At 2:54, you could also use n equals 2 for p equals 3..since you get 48 which is 16 times 3..anyone else see this?
@yuel.5013
@yuel.5013 3 года назад
Indeed, 2^n + 3^n + 6^n - 1 = (-1)^n - 1 mod 3, he just didn't think of the easiest check because he didn't actually look at the equation mod 3, he tried to partially headsolve it, more difficult when you're writing on video at the same time
@ethancheung1676
@ethancheung1676 3 года назад
Since 3^n and 6^n is clearly divisible by 3, what we need is to have 2^n -1 be divisible by 3. It is thus more easily to see 16-1 is the answer than actual multiplying and adding with the 3s and 6s. Yet he chose to follow up with some multiplications which is unnecessary.
@ethancheung1676
@ethancheung1676 3 года назад
@@yuel.5013 good use of the -1. Given that he will use the “inverse” mod quite often afterwards, -1 is more elegant here
@yuel.5013
@yuel.5013 3 года назад
@@ethancheung1676 You're wrong about "easier to see 16-1" because for n = 2 it is 4-1 which is still easier to see, it would have been 4 + 9 + 36 - 1 even if he decided to calc it "the hard way". I got your point though, but I think Michael just complicated things by thinking of actual divisibility lemmas 'cause of headsolving.
@yuel.5013
@yuel.5013 3 года назад
Or shoud I say, livesolving, especially, more than headsolving
@djvalentedochp
@djvalentedochp 3 года назад
top
@littlefermat
@littlefermat 3 года назад
So the trick is simple: Want a natural number? Take an integer and turn it to natural.😎
@nawusayipsunam1643
@nawusayipsunam1643 3 года назад
T
@beanhwak
@beanhwak 3 года назад
For all primes p, p /[2^(p-2)+3^(p-2)+3^(p-2) - 1], hence we can conclude there there are an infinite number of twin primes. Proof: 6(2^-1)= (3)(2)(2^-1) = 3 mod p and 6(3^-1)=(2)(3)(3^-1)= 2 mod p, Hence by subtraction 6^(2^-1)-6(3^-1) = 3-2 = 1, so 6(2^-1-3^-1)=1 Therefore 2^-1 - 3^-1 = 6^-1 (mod p) hence we have shown 2^-1 =3^-1 + 6^-1 Using FLT 2^(p-1) = 1 => 2^(p-2) = 2^-1 (mod p) Similarly 3^(p-1)= 1 => 3^(p-2) = 3^-1 and 6^(p-1) = 6^ -1 => 6^(p-2)=6^-1 (mod p) [2^(p-2) + 3^(p-2) + 3^(p-2) - 1] = [(2^-1+3^-1+6^-1-1] = (2^-1 + 2^-1 - 1] = (2^-1)x2 - 1 = 1 - 1 = 0 (mod p) Therefore p divides 2^(p-2) + 3^(p-2)+6^(p-2) - 1, for all primes p. Therefore there will always be a pair of primes p -2 and p such that p divides 2^(p-2)+3^(p-2)+6^(p-2) - 1 No matter how large p is there exist p - 2 for this to be true. Hence there are INFINITE TWIN PRIMES. Is this not an unsolved problem? Examples:73 divides 2^71 + 3^71 + 6 ^71 and 103 divides 2^101+3^101+6^101 have verified using Wolfram Alpha
@alexrozenbom3430
@alexrozenbom3430 3 года назад
I'm going to be honest. These hints didn't help me.
@rudrasingh2732
@rudrasingh2732 3 года назад
Watch this video: ru-vid.com/video/%D0%B2%D0%B8%D0%B4%D0%B5%D0%BE-yBvsNueXIw4.html
@aekkawinwisitkiatchai7640
@aekkawinwisitkiatchai7640 3 года назад
good place to stop 7:40
@user-A168
@user-A168 3 года назад
Good
@LittleLion72
@LittleLion72 3 года назад
T
Далее
International Math Olympiad | 1960 Q1
14:27
Просмотров 41 тыс.
The smallest such prime...
16:44
Просмотров 56 тыс.
When You're a Chef and a Katana Owner...
00:17
Просмотров 9 млн
Снимать продолжение ?😄
00:46
Просмотров 805 тыс.
UNCRACKABLE? The Collatz Conjecture - Numberphile
7:52
Finding the closed form for a double factorial sum
17:13
A deceivingly difficult differential equation
16:52
Просмотров 245 тыс.
Greek Mathematics Olympiad | 2008 Q2
13:01
Просмотров 45 тыс.
The bridge between number theory and complex analysis
9:59
Simulating the Evolution of Rock, Paper, Scissors
15:00
A team selection number theory problem.
13:41
Просмотров 46 тыс.
The unexpectedly hard windmill question (2011 IMO, Q2)
16:03
An Exact Formula for the Primes: Willans' Formula
14:47
When You're a Chef and a Katana Owner...
00:17
Просмотров 9 млн